2014 dxdy logo

Научный форум dxdy

Математика, Физика, Computer Science, Machine Learning, LaTeX, Механика и Техника, Химия,
Биология и Медицина, Экономика и Финансовая Математика, Гуманитарные науки




Начать новую тему Ответить на тему
 
 Циклическое неравенство
Сообщение30.07.2010, 12:58 


28/03/10
62
для всех положительных $a_1,a_2,...a_n $ доказать неравенство:

$\frac{a_1}{a_1+(n-1)a_2} + \frac{a_2}{a_2+(n-1)a_3} + ... + \frac{a_n}{a_n+(n-1)a_1} \ge 1$ $(n \ge 2)$

 Профиль  
                  
 
 
Сообщение30.07.2010, 15:04 
Заслуженный участник


26/06/07
1929
Tel-aviv
DiviSer в сообщении #341594 писал(а):
для всех положительных $a_1,a_2,...a_n $ доказать неравенство:

$\frac{a_1}{a_1+(n-1)a_2} + \frac{a_2}{a_2+(n-1)a_3} + ... + \frac{a_n}{a_n+(n-1)a_1} \ge 1$ $(n \ge 2)$

Сделаем его симметрическим.
Пусть $\frac{a_1}{a_2}=x_1$, $\frac{a_2}{a_3}=x_2$,..., $\frac{a_n}{a_1}=x_n$, $a_{n+1}=a_1$ и $x_{n+1}=x_1$.
Тогда $\prod\limits_{k=1}^nx_k=1$ и $\sum\limits_{k=1}^n\frac{a_k}{a_k+(n-1)a_{k+1}}\geq1\Leftrightarrow\sum\limits_{k=1}^n\frac{x_k}{x_k+n-1}\geq1\Leftrightarrow$
$\Leftrightarrow\sum\limits_{k=1}^n\left(1-\frac{x_k}{x_k+n-1}\right)\leq n-1\Leftrightarrow\sum\limits_{k=1}^n\frac{1}{x_k+n-1}\right)\leq 1\Leftrightarrow$
$\Leftrightarrow\sum\limits_{k=1}^n\prod\limits_{j\neq k}(x_j+n-1)\leq\prod\limits_{k=1}^n(x_k+n-1)$.
Пусть $\sigma_k$ - элементарный симметрический многочлен от $x_1$, $x_2$,...,$x_n$.
Тогда коэффициент перед $\sigma_k$ в правой части последнего неравенства будет $(n-1)^{n-k}$,
а коэффициент перед $\sigma_k$ в левой его части будет $\frac{n(n-1)^{n-k-1}\binom{n-1}{k}}{\binom{n}{k}}$.
Поскольку $(n-1)^{n-k}\geq\frac{n(n-1)^{n-k-1}\binom{n-1}{k}}{\binom{n}{k}}$ для всех $1\leq k\leq n-1$,
то неравенство очевидным образом следует из AM-GM (ведь $\sigma_k\geq\binom{n}{k}$ и AM-GM не нарушает достижения равенства).

Вот усиление Вашего неравенства для четырёх переменных.
Пусть $a$, $b$, $c$ и $d$ неотрицательные числа такие, что $abc+abd+acd+bcd=4.$ Докажите, что:
$$\frac{1}{a+3}+\frac{1}{b+3}+\frac{1}{c+3}+\frac{1}{d+3}\leq1$$

 Профиль  
                  
 
 Re:
Сообщение31.07.2010, 09:54 


28/03/10
62
arqady в сообщении #341618 писал(а):
DiviSer в сообщении #341594 писал(а):
для всех положительных $a_1,a_2,...a_n $ доказать неравенство:

$\frac{a_1}{a_1+(n-1)a_2} + \frac{a_2}{a_2+(n-1)a_3} + ... + \frac{a_n}{a_n+(n-1)a_1} \ge 1$ $(n \ge 2)$

Сделаем его симметрическим.
Пусть $\frac{a_1}{a_2}=x_1$, $\frac{a_2}{a_3}=x_2$,..., $\frac{a_n}{a_1}=x_n$, $a_{n+1}=a_1$ и $x_{n+1}=x_1$.
Тогда $\prod\limits_{k=1}^nx_k=1$ и $\sum\limits_{k=1}^n\frac{a_k}{a_k+(n-1)a_{k+1}}\geq1\Leftrightarrow\sum\limits_{k=1}^n\frac{x_k}{x_k+n-1}\geq1\Leftrightarrow$
$\Leftrightarrow\sum\limits_{k=1}^n\left(1-\frac{x_k}{x_k+n-1}\right)\leq n-1\Leftrightarrow\sum\limits_{k=1}^n\frac{1}{x_k+n-1}\right)\leq 1\Leftrightarrow$
$\Leftrightarrow\sum\limits_{k=1}^n\prod\limits_{j\neq k}(x_j+n-1)\leq\prod\limits_{k=1}^n(x_k+n-1)$.
Пусть $\sigma_k$ - элементарный симметрический многочлен от $x_1$, $x_2$,...,$x_n$.
Тогда коэффициент перед $\sigma_k$ в правой части последнего неравенства будет $(n-1)^{n-k}$,
а коэффициент перед $\sigma_k$ в левой его части будет $\frac{n(n-1)^{n-k-1}\binom{n-1}{k}}{\binom{n}{k}}$.
Поскольку $(n-1)^{n-k}\geq\frac{n(n-1)^{n-k-1}\binom{n-1}{k}}{\binom{n}{k}}$ для всех $1\leq k\leq n-1$,
то неравенство очевидным образом следует из AM-GM (ведь $\sigma_k\geq\binom{n}{k}$ и AM-GM не нарушает достижения равенства).


как насчет такого неравенства:
$\frac{a_1}{\sqrt{a_1^2+(n^2-1)a_2^2}} + \frac{a_2}{\sqrt{a_2^2+(n^2-1)a_3^2}} + ... + \frac{a_n}{\sqrt{a_n^2+(n^2-1)a_1^2}} \ge 1$ $(n \ge 2)$

arqady в сообщении #341618 писал(а):
Пусть $a$, $b$, $c$ и $d$ неотрицательные числа такие, что $abc+abd+acd+bcd=4.$ Докажите, что:
$$\frac{1}{a+3}+\frac{1}{b+3}+\frac{1}{c+3}+\frac{1}{d+3}\leq1$$

что то схожее с topic24342.html ?

 Профиль  
                  
 
 
Сообщение31.07.2010, 10:27 
Заслуженный участник


26/06/07
1929
Tel-aviv
DiviSer в сообщении #341748 писал(а):
arqady в сообщении #341618 писал(а):
Пусть $a$, $b$, $c$ и $d$ неотрицательные числа такие, что $abc+abd+acd+bcd=4.$ Докажите, что:
$$\frac{1}{a+3}+\frac{1}{b+3}+\frac{1}{c+3}+\frac{1}{d+3}\leq1$$

что то схожее с topic24342.html ?

Нет

 Профиль  
                  
 
 
Сообщение01.08.2010, 22:34 
Заслуженный участник


26/06/07
1929
Tel-aviv
DiviSer в сообщении #341748 писал(а):

как насчет такого неравенства:
$\frac{a_1}{\sqrt{a_1^2+(n^2-1)a_2^2}} + \frac{a_2}{\sqrt{a_2^2+(n^2-1)a_3^2}} + ... + \frac{a_n}{\sqrt{a_n^2+(n^2-1)a_1^2}} \ge 1$ $(n \ge 2)$


Насколько я понял, $a_i>0$.
Пусть $\frac{a_{i+1}^2}{a_i^2}=e^{x_i}$, где $a_{n+1}=a_1$. Тогда $\sum\limits_{i=1}^nx_i=0$ и мы должны доказать, что $\sum\limits_{i=1}^nf(x_i)\geq0$, где $f(x)=\frac{1}{\sqrt{1+(n^2-1)e^x}}$.
Тогда $f''(x)=\frac{(n^2-1)e^x((n^2-1)e^x-2)}{4\left(1+(n^2-1)e^x\right)^{2.5}}$.
Поэтому $f$ выпукла на $\left[\ln{\frac{2}{n^2-1}},+\infty\right)$ и вогнута на $\left(-\infty,\ln{\frac{2}{n^2-1}}\right]$.
Если по меньшей мере два из $x_i$ меньше $\ln{\frac{2}{n^2-1}}$, то $\sum\limits_{i=1}^nf(x_i)>\frac{2}{\sqrt{1+(n^2-1)e^{\ln{\frac{2}{n^2-1}}}}}=\frac{2}{\sqrt3}>1$.
Если все $x_i\in\left[\ln{\frac{2}{n^2-1}},+\infty\right)$ то $\sum\limits_{i=1}^nf(x_i)\geq nf\left(\frac{\sum\limits_{i=1}^nx_i}{n}\right)=nf(0)=1$.
Ну и самый муторный случай, когда ровно один из $x_i$ меньше $\ln{\frac{2}{n^2-1}}$. Обозначим его через $t$. Тогда сумма оставшихся равна $-t$.
Снова применяем неравенство Йенсена к функции $f$ на $\left[\ln{\frac{2}{n^2-1}},+\infty\right)$ и получаем, что остаётся доказать, что
$$\frac{1}{\sqrt{1+(n^2-1)e^t}}+\frac{n-1}{\sqrt{1+(n^2-1)e^{-\frac{t}{n-1}}}}\geq1$$
что проверяется, например, так:
Пусть $e^t=x^{n-1}$, где $0<x<\left(\frac{2}{n^2-1}\right)^{\frac{1}{n-1}}<1$. Тогда остаётся исследовать функцию $g$, где
$g(x)=\frac{1}{\sqrt{1+(n^2-1)x^{n-1}}}+\frac{n-1}{\sqrt{1+\frac{n^2-1}{x}}}$.
Получаем: $g'(x)=\frac{(n-1)^2(n+1)(x^n-1)\left((n^2-1)^3x^{2n-3}-x^n-3(n^2-1)x^{n-1}-1\right)}{2x^2\left((1+(n^2-1)x^{n-1})(1+\frac{n^2-1}{x})\right)^{1.5}\left(x^n\left(1+\frac{n^2-1}{x}\right)^{1.5}+(1+(n^2-1)x^{n-1})^{1.5}\right)}$
Видим, что у $g'$ есть только два положительных корня: $1$ и $x_1$, где $0<x_1<1$ (это для $n\geq3$, а для $n=2$ неравенство легко проверяется).
Поэтому $x_{max}=x_1$, а $g(1)=1$ и поскольку $\lim\limits_{x\rightarrow0^+}g(x)=1$, то неравенство доказано!

 Профиль  
                  
 
 Re: Циклическое неравенство
Сообщение02.08.2010, 18:11 


28/03/10
62
продолжу тему:
для всех $a_1, a_2,...a_n>0$ $(n\geq2)$ доказать следующие нер-ва:

$a_1^{a_1-a_2}+a_2^{a_2-a_3}+...+a_n^{a_n-a_1}\geq n$

$a_1^{a_1}+a_2^{a_2}+...+a_n^{a_n}\geq a_1^{a_2}+a_2^{a_3}+...+a_n^{a_1}$

 Профиль  
                  
Показать сообщения за:  Поле сортировки  
Начать новую тему Ответить на тему  [ Сообщений: 6 ] 

Модераторы: Модераторы Математики, Супермодераторы



Кто сейчас на конференции

Сейчас этот форум просматривают: Bing [bot]


Вы не можете начинать темы
Вы не можете отвечать на сообщения
Вы не можете редактировать свои сообщения
Вы не можете удалять свои сообщения
Вы не можете добавлять вложения

Найти:
Powered by phpBB © 2000, 2002, 2005, 2007 phpBB Group